Reply
 
Thread Tools Display Modes
  #1   Report Post  
Posted to rec.audio.tubes
Rich Sherman
 
Posts: n/a
Default Grid #2 in ultralinear mode

Hello:

I have a question that's been on my mind for several days ever since I
completed a pair of Dynaco clones Mk3 with the MK3 board that Triode
Electronics sells.

Currently I have some SV6550C Power Tubes installed.

The Svetlana spec sheet for these tubes shows that G2 can have a maximum of
400 Volts @ 6 watts.

By connecting these tubes in Push-pull, ultralinear configuration the
voltage on Grid #2 exceeds 400 Volts.

I have not measured the current flowing in the Grid circuit but it must be
much, much less than 15 milliamps so as not to exceed the 6 watt rating of
Grid #2.

Is there something about Ultralinear operation that allows the Grid voltage
to go above maximum? How does let's say operating the tube in Ultralinear
mode compare with tying G2 to the Plate with a 100 Ohm resistor? Are there
any similarities between these two configurations or are they electrically
very different?

The DC resistance of the windings measured between the plate connection and
the ultralinear taps is much lower than 100 Ohms, making the Plate and Grid
voltages probably much closer than strapped in Triode Mode with a 100 Ohm
resistor.

Why do some designers insert a resistor into the Grid circuit in some
Ultralinear amps? Is the purpose of this grid resistor to provide a greater
voltage drop
than if connected directly to the tap on the output transformer primary?

I read that Ultralinear mode behaves somewhere in between Triode mode and
Pentode mode, how? It would seem that without any appreciable voltage drop
between the plate voltage and the G2 voltage the tube is 98% triode
connected?

How is Grid #2 then protected from exceeding it's maximum rating with plate
voltages that are on the order of 490 to 510 Volts DC?

Thanks,
Rich Sherman








  #2   Report Post  
Posted to rec.audio.tubes
Patrick Turner
 
Posts: n/a
Default Grid #2 in ultralinear mode



Rich Sherman wrote:

Hello:

I have a question that's been on my mind for several days ever since I
completed a pair of Dynaco clones Mk3 with the MK3 board that Triode
Electronics sells.

Currently I have some SV6550C Power Tubes installed.

The Svetlana spec sheet for these tubes shows that G2 can have a maximum of
400 Volts @ 6 watts.

By connecting these tubes in Push-pull, ultralinear configuration the
voltage on Grid #2 exceeds 400 Volts.

I have not measured the current flowing in the Grid circuit but it must be
much, much less than 15 milliamps so as not to exceed the 6 watt rating of
Grid #2.

Is there something about Ultralinear operation that allows the Grid voltage
to go above maximum? How does let's say operating the tube in Ultralinear
mode compare with tying G2 to the Plate with a 100 Ohm resistor? Are there
any similarities between these two configurations or are they electrically
very different?


The screen rating for maximum dissipation may be 6 watts, which means that if
g2 is at 400V, and Ig2 = about 5% of Ia, so about 2.5mA at idle, then pd g2 = 1
watt.

However, g2 current changes under signal handling conditions,
and the g2 I x V product will increase over the idle conditions.

The 6 watt pd g2 is based on using the g2 tied to a fixed voltage supply
known as beam tetrode operation. In this mode the screen has the highest
dissipation,
ie, volts x amps is greatest during dynamic signal conditions.

When the anode voltage falls when it conducts high load current, Eg2 remains at
the high fixed voltage
and attracts many more electrons than when it is connected to the anode when
triode operation
is employed.

So triode operation during dynamic conditions with signals gives the lowest g2
dissipation.
Triode operation anode voltage swings are usually lower than beam tetrode.
UL operation is between triode and beam tet.




The DC resistance of the windings measured between the plate connection and
the ultralinear taps is much lower than 100 Ohms, making the Plate and Grid
voltages probably much closer than strapped in Triode Mode with a 100 Ohm
resistor.


The 100ohm R does little except control parasitic HF oscillations.
Indeed Va and Vg2 are almost identical.



Why do some designers insert a resistor into the Grid circuit in some
Ultralinear amps? Is the purpose of this grid resistor to provide a greater
voltage drop


The R = typically 2k2 is to stop RF oscillations on the grid1
the 100hm R for grid2, the screen is also tom prevent RF generations.


than if connected directly to the tap on the output transformer primary?

I read that Ultralinear mode behaves somewhere in between Triode mode and
Pentode mode, how? It would seem that without any appreciable voltage drop
between the plate voltage and the G2 voltage the tube is 98% triode
connected?


With g2 connected via 100ohms to the anode, it *is* triode operation.
The screen delivers the voltage at the anode to the electron stream as local
NFB.
When UL is used, if the tap is at 50% along the 1/2 primary winding, then
the electon stream has only 50% of the anode voltage applied as NFB.
But the effect is to reduce Ra and thd/imd.
With g2 taken to a fixed B+, very little NFB is allowed to affect the electron
stream from the g2.
The screen, g2, is a kind of control grid like grid1, but it has less
transconductance.



How is Grid #2 then protected from exceeding it's maximum rating with plate
voltages that are on the order of 490 to 510 Volts DC?


Because UL op with UL taps over 30% and up to triode operation
which is basically 100% UL op result in lower dissipation so the Eg2 and Ig2
product are lower
with signals and UL operation up to Ea = Eg2 = 550V is possible.

Try reading some old books about basic tube operation.

Patrick Turner.



Thanks,
Rich Sherman


  #3   Report Post  
Posted to rec.audio.tubes
Ian Iveson
 
Posts: n/a
Default Grid #2 in ultralinear mode


Rich Sherman wrote

I have a question that's been on my mind for several days ever
since I
completed a pair of Dynaco clones Mk3 with the MK3 board that
Triode
Electronics sells.

Currently I have some SV6550C Power Tubes installed.

The Svetlana spec sheet for these tubes shows that G2 can have a
maximum of
400 Volts @ 6 watts.

By connecting these tubes in Push-pull, ultralinear configuration
the
voltage on Grid #2 exceeds 400 Volts.

I have not measured the current flowing in the Grid circuit but it
must be
much, much less than 15 milliamps so as not to exceed the 6 watt
rating of
Grid #2.

Is there something about Ultralinear operation that allows the
Grid voltage
to go above maximum? How does let's say operating the tube in
Ultralinear
mode compare with tying G2 to the Plate with a 100 Ohm resistor?
Are there
any similarities between these two configurations or are they
electrically
very different?


Yes, and the same for triode mode. This series of beam tetrodes
states that Vs should be less than Va but AFAIK that's for pentode
mode. As long as screen current limitations are observed, it should
be OK.

Does anyone know exactly *why* some beam tetrodes state this
limitation, in addition to usual screen current considerations?

In comparing modes you must consider AC conditions, not the DC
operating point, which is the same in each case. In triode mode the
screen sees the same AC voltage as the anode, in pentode mode it
sees zero AC voltage, and at a tapping of X percent, the screen sees
X percent of the AC on the anode.

The DC resistance of the windings measured between the plate
connection and
the ultralinear taps is much lower than 100 Ohms, making the Plate
and Grid
voltages probably much closer than strapped in Triode Mode with a
100 Ohm
resistor.

Why do some designers insert a resistor into the Grid circuit in
some
Ultralinear amps? Is the purpose of this grid resistor to provide
a greater
voltage drop
than if connected directly to the tap on the output transformer
primary?


You mean screen, G2? Two reasons. First is to discourage parasitic
oscillation, second is a precaution against heavy screen current at
high signal amplitudes when Va falls to a very low value (then
electrons can't be arsed to clamber all the way to the anode). The
downside is that screen current then is extremely non-linear, so
converting it to a voltage using a resistor results in a very
distorted voltage.

I read that Ultralinear mode behaves somewhere in between Triode
mode and
Pentode mode, how? It would seem that without any appreciable
voltage drop
between the plate voltage and the G2 voltage the tube is 98%
triode
connected?

How is Grid #2 then protected from exceeding it's maximum rating
with plate
voltages that are on the order of 490 to 510 Volts DC?


As long as electrons have enough energy to reach the anode, you are
OK. Have a look at the characteristic curves to see how screen
current soars at low anode voltages in pentode mode. Not so true in
triode mode because screen voltage is also low at the same time, so
there is little current anyway. Ultralinear pro rata, if we forget
about phase error.

cheers, Ian


  #4   Report Post  
Posted to rec.audio.tubes
Chris Hornbeck
 
Posts: n/a
Default Grid #2 in ultralinear mode

On Fri, 20 Jan 2006 16:47:31 GMT, "Ian Iveson"
wrote:

Yes, and the same for triode mode. This series of beam tetrodes
states that Vs should be less than Va but AFAIK that's for pentode
mode. As long as screen current limitations are observed, it should
be OK.

Does anyone know exactly *why* some beam tetrodes state this
limitation, in addition to usual screen current considerations?


When G2 has a higher voltage than the plate, it begins
to get ambitions of actually *being* the plate. Havoc,
wailing, and gnashing of teeth ensue.

Chris Hornbeck
  #5   Report Post  
Posted to rec.audio.tubes
Phil Allison
 
Posts: n/a
Default Grid #2 in ultralinear mode


"Rich Sherman"

Currently I have some SV6550C Power Tubes installed.

The Svetlana spec sheet for these tubes shows that G2 can have a maximum
of
400 Volts @ 6 watts.

By connecting these tubes in Push-pull, ultralinear configuration the
voltage on Grid #2 exceeds 400 Volts.

I have not measured the current flowing in the Grid circuit but it must be
much, much less than 15 milliamps so as not to exceed the 6 watt rating of
Grid #2.



** You exaggerate - the idle current flow will be need to be less than 15
mA, but not by very much.

The actual value depends on the tube bias current - so the G1 voltage.


Is there something about Ultralinear operation that allows the Grid
voltage
to go above maximum?


** Yep.


How does let's say operating the tube in Ultralinear
mode compare with tying G2 to the Plate with a 100 Ohm resistor?



** That is even kinder to G2 - since then it will NEVER have a voltage
that EXCEEDS that of the plate at any time during a cycle.


Why do some designers insert a resistor into the Grid circuit in some
Ultralinear amps? Is the purpose of this grid resistor to provide a
greater
voltage drop than if connected directly to the tap on the output
transformer
primary?



** Yep, particularly when the amp is driven into clipping.


I read that Ultralinear mode behaves somewhere in between Triode mode and
Pentode mode, how? It would seem that without any appreciable voltage drop
between the plate voltage and the G2 voltage the tube is 98% triode
connected?



** There is a very substantial difference between plate and G2 voltages
during AC operation at full power.

Eg: At the peak of the wave, the plate voltage will be less than 100 volts
while the screen will be circa 350 volts ( for a 500 volt supply.)


How is Grid #2 then protected from exceeding it's maximum rating with
plate
voltages that are on the order of 490 to 510 Volts DC?



** Consider AC operation.

Imagine how the screen voltage partially follows the plate voltage down as
tube current increases then up as it reduces.

So overall, the screen is happier ( ie less dissipation) in UL mode than in
pentode mode.

The *worst case scenario* is when a PP pentode output stage is drive into
hard clipping - then the plate is at a low voltage and the screen at high
one when the tube is conducting maximum current. It is only by using high
value ( 1 or 2 kohms) screen resistors that saves the tube from sudden
failure OR ( and much better) by using a reduced screen supply compared
to the plate supply.



........ Phil





  #6   Report Post  
Posted to rec.audio.tubes
Ian Iveson
 
Posts: n/a
Default Grid #2 in ultralinear mode

Chris Hornbeck wrote

Yes, and the same for triode mode. This series of beam tetrodes
states that Vs should be less than Va but AFAIK that's for pentode
mode. As long as screen current limitations are observed, it
should
be OK.


Does anyone know exactly *why* some beam tetrodes state this
limitation, in addition to usual screen current considerations?


When G2 has a higher voltage than the plate, it begins
to get ambitions of actually *being* the plate. Havoc,
wailing, and gnashing of teeth ensue.


Well, according to my relabelled models, G2 *is* the anode, so my
electrodes must have different ambitions :-)

My drain is thwarted in its efforts to attract shagged-out
electrons. Errant behaviour is thus a result of failure rather than
achievement.

*My* electrodes are always well-intentioned.

Continuing however with the usual nomenclature...

What I mean to ask is why one particular family of beam tetrodes has
a screen voltage limit so much lower than that of the anode. This
trait of the 6L6 has prompted loads of questions over the years. The
screen mu is not specially remarkable, so presumably it is a normal
kind of distance from the cathode. So I wonder if the anode is a
long way from the screen, or if the screen is made of very thin
wire, or what?

cheers, Ian



  #7   Report Post  
Posted to rec.audio.tubes
mick
 
Posts: n/a
Default Grid #2 in ultralinear mode

On Sat, 21 Jan 2006 11:25:14 +0000, Ian Iveson burbled:

Chris Hornbeck wrote

Yes, and the same for triode mode. This series of beam tetrodes states
that Vs should be less than Va but AFAIK that's for pentode mode. As
long as screen current limitations are observed, it should
be OK.


Does anyone know exactly *why* some beam tetrodes state this limitation,
in addition to usual screen current considerations?


When G2 has a higher voltage than the plate, it begins to get ambitions
of actually *being* the plate. Havoc, wailing, and gnashing of teeth
ensue.


Well, according to my relabelled models, G2 *is* the anode, so my
electrodes must have different ambitions :-)

My drain is thwarted in its efforts to attract shagged-out electrons.
Errant behaviour is thus a result of failure rather than achievement.

*My* electrodes are always well-intentioned.

Continuing however with the usual nomenclature...

What I mean to ask is why one particular family of beam tetrodes has a
screen voltage limit so much lower than that of the anode. This trait of
the 6L6 has prompted loads of questions over the years. The screen mu is
not specially remarkable, so presumably it is a normal kind of distance
from the cathode. So I wonder if the anode is a long way from the screen,
or if the screen is made of very thin wire, or what?


In a 6L6 g1 and g2 are wire spirals wound in the same direction and
in-line, allowing a straight line through from the cathode, through
the grids, between the beam-forming plates to the anode. I don't know
about the diameter of the wire used, but it could be an indication of why
the max g2 dissipation figure given in the spec is quite low.

This form of construction *could* be why 6L6s seem to be quite good when
triode-connected. In a normal tetrode or pentode the electron stream will
hit g2 or g3, causing dissipation in those grids. g2 does operate as an
anode, with the true anode merely being an electron collector. The 6L6, on
the other hand, still uses the true anode as the electrons will mostly
tend to miss g2, and there is no g3 to get in the way.

--
Mick
(no M$ software on here... :-) )
Web: http://www.nascom.info


  #8   Report Post  
Posted to rec.audio.tubes
Phil Allison
 
Posts: n/a
Default Grid #2 in ultralinear mode


"mick"

In a 6L6 g1 and g2 are wire spirals wound in the same direction and
in-line, allowing a straight line through from the cathode, through
the grids, between the beam-forming plates to the anode.



** Same construction method used in beam tubes and power pentodes like the
EL34 and EL84.

The screen wires are sheltered " down wind " of G1 where electron density is
low.


I don't know
about the diameter of the wire used, but it could be an indication of why
the max g2 dissipation figure given in the spec is quite low.



** Heat radiated from G2 is all collected by the anode anyhow. At least in
UL mode, the AC component of screen current adds to the output power -
unlike pentode mode where it is entirely wasted.


This form of construction *could* be why 6L6s seem to be quite good when
triode-connected. In a normal tetrode or pentode the electron stream will
hit g2 or g3, causing dissipation in those grids. g2 does operate as an
anode, with the true anode merely being an electron collector.



** Huh ?

" with the true anode merely being an electron collector "

What drivel.



The 6L6, on
the other hand, still uses the true anode as the electrons will mostly
tend to miss g2, and there is no g3 to get in the way.




** Make up your mind.

Compewter geeks........



........ Phil


  #9   Report Post  
Posted to rec.audio.tubes
Patrick Turner
 
Posts: n/a
Default Grid #2 in ultralinear mode



Phil Allison wrote:

"mick"

In a 6L6 g1 and g2 are wire spirals wound in the same direction and
in-line, allowing a straight line through from the cathode, through
the grids, between the beam-forming plates to the anode.


** Same construction method used in beam tubes and power pentodes like the
EL34 and EL84.

The screen wires are sheltered " down wind " of G1 where electron density is
low.


Screen grid wires are placed so most electrons miss hitting the screen;
hence screen current is below 10% of the total anode and screen currents.

The screen inserts the presence of a fixed voltage field between g1 and the
anode,
the screen has a mighty presence, but is not designed to absorb
many electrons like the anode.



I don't know
about the diameter of the wire used, but it could be an indication of why
the max g2 dissipation figure given in the spec is quite low.


** Heat radiated from G2 is all collected by the anode anyhow. At least in
UL mode, the AC component of screen current adds to the output power -
unlike pentode mode where it is entirely wasted.


The output power contribution by the screen to OPT primary is tiny
compared to the anode contribution.
UL power output is usually 10% or more less than pentode or beam terode
connection,
and the power "wasted" at the screen is also negligible. The "wasted"
screen power is really an investment of power to maximise plate power.




This form of construction *could* be why 6L6s seem to be quite good when
triode-connected. In a normal tetrode or pentode the electron stream will
hit g2 or g3, causing dissipation in those grids. g2 does operate as an
anode, with the true anode merely being an electron collector.


** Huh ?

" with the true anode merely being an electron collector "

What drivel.


Its not quite as drivelly as you think.

If you had a 6L6 with a plate structure which was the same diameter as the
screen grid winding, and there was no screen, then the tube would act
exactly like a 6L6 in triode, but it would be a real triode.
But with a real 6L6 strapped in triode, the screen voltage acts to inject a
plate voltage closer to the
control grid and cathode
than where the plate is actually located, which is further out beyond the
screen,
so the anode can be a larger diameter structure, and this possibly allows
slightly higher dissipation for a given amount of plate metal.



The 6L6, on
the other hand, still uses the true anode as the electrons will mostly
tend to miss g2, and there is no g3 to get in the way.


** Make up your mind.

Compewter geeks........


Tube operation is messy.

Electrons swirl around the cathode in a cloud like a swarm of bees,
and they are emitted and reabsorbed constantly by the cathode.
The electrons can slightly feel the presence of the +ve charges on the far side
of the control grid1,
and if g1 goes a bit less -ve, ie, swings slightly more +ve, then some sneak
past g1,
and accelerate fast towards whatever +ve charge is beyond.
Most miss hitting g2, and continue on towads whatever else is positive,
which is the anode, and they belt into it generating the heat
= Ea x Ia in watts.
But quite a few bounce off the anode because they didn't find a soft landing
into orbit on an atom to fly into.
This is secondary emission. But in pentodes a suppressor grid is fitted,
and its charge is at 0V, and it repels most of the electrons bouncing back from
the anode towards the g2
and with much reduced speed, so they are forced to return to the anode where
eventually they are absorbed.

The beam forming plates in a beam tetrode
repel electrons on two sides of the electron flow and squeeze the broad stream
that occurs in a pentode into
a narrow beam and thus creating a space charge between the g2 and anode
that suppresses the secondary emission by forcing bouncing low velocity
electrons back towards the anode.

Patrick Turner.




....... Phil


  #10   Report Post  
Posted to rec.audio.tubes
Phil Allison
 
Posts: n/a
Default **** OFF Tureneroid Scum !!


" Patrick Turneroid = Autodidact & Imbecile "

( the man with one too many chromosomes )



** I do **NOT ** want any demented, lying, autistic, criminal, stalking
****wits "urinating" on my posts like some incontinent smelly mutt piddling
on every light pole it sees.

THIS MEANS YOU TURNER !!!!!!!!





......... Phil




  #11   Report Post  
Posted to rec.audio.tubes
Patrick Turner
 
Posts: n/a
Default **** OFF Tureneroid Scum !!



Phil Allison wrote:

" Patrick Turneroid = Autodidact & Imbecile "

( the man with one too many chromosomes )

** I do **NOT ** want any demented, lying, autistic, criminal, stalking
****wits "urinating" on my posts like some incontinent smelly mutt piddling
on every light pole it sees.

THIS MEANS YOU TURNER !!!!!!!!

........ Phil


Phil, I will not **** off at all.

I will if necessary read the riot act to you
syllable by syllable to make your life as miserable as you
deliberately try to make others' if and when it suits me.

This is a public forum, not a private one, and I have an inalienable right to
post replies as ever I see fit to all the *WHOLE* group, where you are just a
member, like me.
You don't own your own posts once they are public.
You have no rights or powers to stop ppl posting to what you have said.
This is the World, 2006, not Germany, 1936.
So be tolerant and polite Phil, its a better way than spitting the dummy like a
2 yr old!

Now the facts are that you made a quite polite and respectable post
to which I replied in kind without undue insult.

Your reaction???

To insult me and post an entire suite of total lies about my net behaviour.

So Phil, get a life, take your anti bipolar syndrome pills.

And it wasn't me that stalked you before last christmas at aus.hi-fi
for 3 months and then tried to make unwanted phone calls.
It was you that did all the searching to dig up **** and phone me with the
intent of
harrassment.
Then you posted all that utter bull**** about the content of a 40 second
phone call we allegedly had, and for 2 mths you said you'd taped it, but when I
insisted you
make the taped call available to the public, you couldn't/wouldn't, and your
credibility on personal matters lies in the gutter, lower than a snake's arsole.

So don't make stoopid claims Phil, you cannot get away with them with me.

Stop being an idiotic hypocrite.

Patrick Turner.

  #12   Report Post  
Posted to rec.audio.tubes
Phil Allison
 
Posts: n/a
Default **** OFF Tureneroid Scum !!


" Patrick Turneroid = Autodidact & Imbecile & Criminal "

( the man with one too many chromosomes )


** I do **NOT ** want any demented, lying, autistic, criminal, stalking
****wits "urinating" on my posts like some incontinent smelly mutt piddling
on every light pole it sees.

THIS MEANS YOU TURNER !!!!!!!!



( snip usual pile of criminal defamation and INSANE Turneroid lies )


** Turner is truly evil.

Turner is a lying, psychopathic pile of sub human ****.

Turner is also an autistic cretin & public menace.

Of course, mental defects and deficiencies are the cause of but constitute
NO excuse for his criminal behaviour.

May the SOB long rot in hell.





......... Phil



  #13   Report Post  
Posted to rec.audio.tubes
RdM
 
Posts: n/a
Default Grid #2 in ultralinear mode

Patrick Turner wrote in
. au:

Electrons swirl around the cathode in a cloud like a swarm of bees,
and they are emitted and reabsorbed constantly by the cathode.


Reabsorbed? By what process?
The heated cathode, boiling off electrons in a vacuum, is also reabsorbing
them, you say? What percentage of emitted electrons would you say are
reabsorbed, thus reducing both the anode current and available electrons in
the g-k space cloud? I don't think I've heard of this before ...
  #14   Report Post  
Posted to rec.audio.tubes
mick
 
Posts: n/a
Default Grid #2 in ultralinear mode

On Sun, 22 Jan 2006 11:17:07 +1100, Phil Allison burbled:


"mick"

In a 6L6 g1 and g2 are wire spirals wound in the same direction and
in-line, allowing a straight line through from the cathode, through the
grids, between the beam-forming plates to the anode.



** Same construction method used in beam tubes and power pentodes like the
EL34 and EL84.

The screen wires are sheltered " down wind " of G1 where electron density
is low.


I understood that EL84 and EL34 were true pentodes? If so, surely g3 only
has a few turns as it isn't a normal control grid. Unless it has the same
number of turns as g1 and g2 the spiral has to cross the electron stream
and will have more screen current and dissipate more power. Which figures
- they have a higher g3 power dissipation rating than the 6L6.


** Heat radiated from G2 is all collected by the anode anyhow. At least in
UL mode, the AC component of screen current adds to the output power -
unlike pentode mode where it is entirely wasted.


The 6L6 was designed to have a very low screen current - it was designed
for efficiency. g2 is actually quite close to g1 in a 6L6. It can't get
rid of heat to the anode easily can it?


This form of construction *could* be why 6L6s seem to be quite good when
triode-connected. In a normal tetrode or pentode the electron stream
will hit g2 or g3, causing dissipation in those grids. g2 does operate
as an anode, with the true anode merely being an electron collector.



** Huh ?

" with the true anode merely being an electron collector "

What drivel.


If g2 is at the same potential as the anode (which it is when triode
connected) then that's where a lot of electrons will end up. It has to act
as an anode. The true anode will only collect electrons that manage to
escape through g2 and g3 - and g3 is at cathode potental so the electrons
will tend to slow down and possibly fall back to g2. This will definitely
happen in valves that arn't grid-aligned but less so where the grids are
aligned.

The 6L6, on
the other hand, still uses the true anode as the electrons will mostly
tend to miss g2, and there is no g3 to get in the way.




** Make up your mind.


Did... :-) 6L6 has beam plates instead of g3, so the electrons gave
a clear run from g2 to anode & tend not to hit them. g2 adds a lot of
acceleration, but is close to g1 & aligned so electrons tend to escape
through it.

Compewter geeks........


/me grins & goes back to pondering turing machines...

Cheers, Phil!
--
Mick
(no M$ software on here... :-) )
Web: http://www.nascom.info


  #15   Report Post  
Posted to rec.audio.tubes
RdM
 
Posts: n/a
Default Grid #2 in ultralinear mode

"Phil Allison" wrote in
:

** Heat radiated from G2 is all collected by the anode anyhow.


Some heat "radiated", but I bet a fair bit is dissipated through conduction
via the rods holding the spiral - and any little extra fins atop them as well.


  #16   Report Post  
Posted to rec.audio.tubes
mick
 
Posts: n/a
Default Thanks, Phil! I needed that... ;-)

On Sun, 22 Jan 2006 22:32:55 +1100, Phil Allison burbled:


"mick"

Jesus Christ !!!!!!!!!!!!!!!!

.. yet another, autistic, pommy, ****wit know nothing !!!.



I understood that EL84 and EL34 were true pentodes? If so, surely g3
only has a few turns as it isn't a normal control grid.




** PUBLIC WARNING TO MICK !!!


NEVER EVER POST on RAT again !

Or else I will ** FORCE ** you to **** off.



I *promise* that will be an extremely painful experience.

Autistic compewter ****s like you need to be publicly castrated.

With a blunt carving knife.



grin

I knew I could depend on you, Phil.

lol!


--
Mick
(no M$ software on here... :-) )
Web: http://www.nascom.info


  #17   Report Post  
Posted to rec.audio.tubes
Phil Allison
 
Posts: n/a
Default "mick" gets his BALLS cut OFF

"mick"


** Jesus ****ing Christ !!!!!!!!!!!!!!!!

.. yet another, autistic, pommy, ****wit know nothing **** !!!




I understood that EL84 and EL34 were true pentodes? If so, surely g3 only
has a few turns as it isn't a normal control grid.



** PUBLIC WARNING TO MICK !!!


NEVER EVER POST on RAT again !

Or else I will ** FORCE ** you to **** off.


I *promise* that will be an extremely painful experience.

Autistic compewter ****s like you need to be publicly castrated.

With a blunt carving knife.

Much blood.

Much pain.






......... Phil



  #18   Report Post  
Posted to rec.audio.tubes
Patrick Turner
 
Posts: n/a
Default Grid #2 in ultralinear mode



mick wrote:

On Sun, 22 Jan 2006 11:17:07 +1100, Phil Allison burbled:


"mick"

In a 6L6 g1 and g2 are wire spirals wound in the same direction and
in-line, allowing a straight line through from the cathode, through the
grids, between the beam-forming plates to the anode.



** Same construction method used in beam tubes and power pentodes like the
EL34 and EL84.

The screen wires are sheltered " down wind " of G1 where electron density
is low.


I understood that EL84 and EL34 were true pentodes? If so, surely g3 only
has a few turns as it isn't a normal control grid.


That is correct generally with pentodes; the field density required to
give suppression of secondary emisssion is fairly weak; it only needs to be
strong enough and no more.
However, with pentodes like the 6DT6 small 7 pin signal pentode the suppressor
grid
has considerable ability to control the electron flow and this makes it an
excellent tube to use
as a quadrature FM detector in FM radios and TV sets.


Unless it has the same
number of turns as g1 and g2 the spiral has to cross the electron stream
and will have more screen current and dissipate more power. Which figures
- they have a higher g3 power dissipation rating than the 6L6.


There is no g3 in a 6L6, but beam forming plates.
The power dissipated in g3 is tiny.

I suggest you make some measurements of the current flow into and out of all the
tube electrodes
of EL34 and 6L6.

Gently smash open a tube to examine how its made.
( break the glass while holding the tube in a water filled bucket, since tubes
have barium and strontium within and clean up carefully. The water won't wreck
the
while you examine them and will prevent dust wafting up your nose
and into your lungs. )





** Heat radiated from G2 is all collected by the anode anyhow. At least in
UL mode, the AC component of screen current adds to the output power -
unlike pentode mode where it is entirely wasted.


The 6L6 was designed to have a very low screen current - it was designed
for efficiency. g2 is actually quite close to g1 in a 6L6. It can't get
rid of heat to the anode easily can it?


The g1 and g2 temperature will be slightly lower than the anode although under
fault conditions the g2 T will rise,
maybe it turns red, orange, yellow, then melts....



This form of construction *could* be why 6L6s seem to be quite good when
triode-connected. In a normal tetrode or pentode the electron stream
will hit g2 or g3, causing dissipation in those grids. g2 does operate
as an anode, with the true anode merely being an electron collector.



** Huh ?

" with the true anode merely being an electron collector "

What drivel.


If g2 is at the same potential as the anode (which it is when triode
connected) then that's where a lot of electrons will end up.


Try measuring the g2 current in a triode connected tube.
Its never a huge % of plate current

It has to act
as an anode.


It does act "as an anode" in the sense that it supplies an electric voltage
field
that acts as if the anode were the same size and in the same position as the g2.

But electrons mainly miss hitting the g2, and go on to the real anode.



The true anode will only collect electrons that manage to
escape through g2 and g3 - and g3 is at cathode potental so the electrons
will tend to slow down and possibly fall back to g2.


The g3 has a negligible effect in slowing down electrons that have made it past
g2
but g3 creates just enough of a field at much lower potential to
repel most of the secondary emission electrons bouncing back from the anode.

This will definitely
happen in valves that arn't grid-aligned but less so where the grids are
aligned.


Alignment is important, and where alignment is poor the g2 current is much
greater than normal
because the g2 wires are not in the "shadow" of g1 grid wires.




The 6L6, on
the other hand, still uses the true anode as the electrons will mostly
tend to miss g2, and there is no g3 to get in the way.




** Make up your mind.


Did... :-) 6L6 has beam plates instead of g3, so the electrons gave
a clear run from g2 to anode & tend not to hit them. g2 adds a lot of
acceleration, but is close to g1 & aligned so electrons tend to escape
through it.


g3 currents are tiny, since the electrons absorbed by g3 is tiny, ditto for beam
forming
plates, and the plates are usually always tied to the cathode inside the tube.

Patrick Turner.



Compewter geeks........


/me grins & goes back to pondering turing machines...

Cheers, Phil!
--
Mick
(no M$ software on here... :-) )
Web: http://www.nascom.info


  #19   Report Post  
Posted to rec.audio.tubes
Patrick Turner
 
Posts: n/a
Default Grid #2 in ultralinear mode



RdM wrote:

Patrick Turner wrote in
. au:

Electrons swirl around the cathode in a cloud like a swarm of bees,
and they are emitted and reabsorbed constantly by the cathode.


OK, try reading the old books about thermionic emissions from cathodes.

From what i read electrons are emitted if the cathode temp and material is just
right,
and so what happens to an electron once emitted?

They have an escape velocity, but do they hover in space near the cathode?
They are vigourously on the move all the time, coming and going, and the
cloud density is slightly affected by the anode or screen voltages, and a lot
affected by the
g1 voltage; with a +ve going grid, electron stream away from the cathode, so
the
*net* emission increases, and it is a substantially linear phenomena.



Reabsorbed? By what process?
The heated cathode, boiling off electrons in a vacuum, is also reabsorbing
them, you say?


Nothing is "boiled off", but rather the electrons are given enough heat energy
to
spin off atoms, breaking the force which attracts them to the cathode.
They slow down, and return, become re energised...
Its like bees in a swarm, some are leaving and some come back, so a net
effect is a cloud of bees on the move.


What percentage of emitted electrons would you say are
reabsorbed, thus reducing both the anode current and available electrons in
the g-k space cloud? I don't think I've heard of this before ...


I am not a tube making expert, nor a molecular scientist, am a lay person
interested in tube
usage in tube circuits.

I have some mental idea about the electron rich cloud that surrounds a cathode
in a tube.

Without any plate current if g1 in negative enough, there is still a cloud of
electrons
coming and going from the cathode and acting in a state of equilibrium.

The cloud builds up to limit the density of electrons.
The speed at which changes to emission from the cathode can be effected is
extraordinarily
fast, allowing extremely high frequencies to be amplified.

The capacity for increasing the emissions from the idle state is usually many
times,
and a 300B cathode can emit an amp for a short time if the grid is made quite
positive.
But usually, the emission of 70mA +/- 60mA load current change is well within
the abilities
of the cathode to emit electrons so that the tube is substantially linear.
The negative charge held by the cloud of electrons around the cathode repel
electrons to some extent and prevent the charge build up to exceed a point of
equilibrium.

Of course things get more mysterious where you have a tube loaded by a CCS,
then *NO CURRENT* change occurs, and the tube operates as a pure voltage
division device.

Patrick Turner.




  #20   Report Post  
Posted to rec.audio.tubes
Ian Iveson
 
Posts: n/a
Default Grid #2 in ultralinear mode

Patrick Turner wrote in
. au:

Electrons swirl around the cathode in a cloud like a swarm of
bees,
and they are emitted and reabsorbed constantly by the cathode.


Reabsorbed? By what process?
The heated cathode, boiling off electrons in a vacuum, is also
reabsorbing
them, you say? What percentage of emitted electrons would you say
are
reabsorbed, thus reducing both the anode current and available
electrons in
the g-k space cloud? I don't think I've heard of this before ...


And why, when the heater is on but all electrodes disconnected,
doesn't the bottom of the valve fill up with electrons that have
fallen from the cathode?

The boiling analogy is poor because water doesn't fall back into the
kettle. A valve would be a voltage generator. A deepish layer of
peas on a drum would be a better model. Kinetic v potential energy,
dynamic equilibrium (I guess some clots would call that feedback).

Magnetic fields should also be considered I guess. Some talk of
"wandering" and "swirling", like smoke or steam rushing past
obstacles. Obviously that is nonsense. There is dynamic interaction
of magnetic and electric fields.

How fast do electrons pass through a vacuum? If all this movement
happens instantaneously, then it's not movement, is it?
"Instantaneous" and "dynamic" don't sit well together. Bees can't
fly at the speed of light because their wings would need to flap
even faster.

Ultimately, whether the equilibrium is static or dynamic depends on
how you think about it, I guess.

cheers, Ian





  #21   Report Post  
Posted to rec.audio.tubes
mick
 
Posts: n/a
Default Grid #2 in ultralinear mode

On Sun, 22 Jan 2006 12:58:41 +0000, Patrick Turner burbled:



mick wrote:

On Sun, 22 Jan 2006 11:17:07 +1100, Phil Allison burbled:


"mick"

In a 6L6 g1 and g2 are wire spirals wound in the same direction and
in-line, allowing a straight line through from the cathode, through
the grids, between the beam-forming plates to the anode.


** Same construction method used in beam tubes and power pentodes like
the EL34 and EL84.

The screen wires are sheltered " down wind " of G1 where electron
density is low.


I understood that EL84 and EL34 were true pentodes? If so, surely g3
only has a few turns as it isn't a normal control grid.


That is correct generally with pentodes; the field density required to
give suppression of secondary emisssion is fairly weak; it only needs to
be strong enough and no more.
However, with pentodes like the 6DT6 small 7 pin signal pentode the
suppressor grid
has considerable ability to control the electron flow and this makes it an
excellent tube to use
as a quadrature FM detector in FM radios and TV sets.

more good stuff snipped

Thanks Patrick, that has helped a lot. :-)

--
Mick
(no M$ software on here... :-) )
Web: http://www.nascom.info


  #22   Report Post  
Posted to rec.audio.tubes
mick
 
Posts: n/a
Default Still winding Phil up.... :-)

On Sun, 22 Jan 2006 23:20:37 +1100, Phil Allison burbled:

"mick"


** Jesus ****ing Christ !!!!!!!!!!!!!!!!

.. yet another, autistic, pommy, ****wit know nothing **** !!!




I understood that EL84 and EL34 were true pentodes? If so, surely g3
only has a few turns as it isn't a normal control grid.



** PUBLIC WARNING TO MICK !!!


NEVER EVER POST on RAT again !

Or else I will ** FORCE ** you to **** off.


I *promise* that will be an extremely painful experience.

Autistic compewter ****s like you need to be publicly castrated.

With a blunt carving knife.

Much blood.

Much pain.



Thanks Phil. The feeling is mutual sometimes.
;-)

--
Mick
(no M$ software on here... :-) )
Web: http://www.nascom.info


  #23   Report Post  
Posted to rec.audio.tubes
Ian Iveson
 
Posts: n/a
Default Grid #2 in ultralinear mode

Patrick Turner wrote

Electrons swirl around the cathode in a cloud like a swarm of
bees,
and they are emitted and reabsorbed constantly by the cathode.


And positive electricity makes flowers grow...

OK, try reading the old books about thermionic emissions from
cathodes.


Why old? They didn't know so much then.

Anyway, it should be obvious that you could reply to *any* question
here with "read a book". Equally, you may assume that most ppl know
what a book is, and what it is for. Hence you may conclude that
"read a book" will but rarely be a useful answer, particularly if
you don't say which parts of which book.

Anyway, if you recite books all day, you might expect ppl will have
less incentive to read them.

I was rather hoping a living nuclear scientist might respond...fresh
insight is not an impossibility.

What are you here for?

...They have an escape velocity,...


No, that's just what they don't have. Otherwise they would escape
:-(

I was told at school that electrons in metals tend to have affinity
to the bulk, rather than to particular atoms. Like a sea of
electrons in an ion matrix. The atomic forces don't take
much escaping...although I suppose you could have a notion of an
escape velocity for the atomic forces, and then another for the
electrostatic forces. They achieve the former but not the latter.

None of which takes into account the surface structure of the metal,
which is different from that within.

cheers, Ian



  #24   Report Post  
Posted to rec.audio.tubes
Ruud Broens
 
Posts: n/a
Default Grid #2 in ultralinear mode


"Patrick Turner" wrote in message
...
:
:
: RdM wrote:
:
: Patrick Turner wrote in
: . au:
:
: Electrons swirl around the cathode in a cloud like a swarm of bees,
: and they are emitted and reabsorbed constantly by the cathode.
:
: OK, try reading the old books about thermionic emissions from cathodes.
:
: From what i read electrons are emitted if the cathode temp and material is just
: right,
: and so what happens to an electron once emitted?
:
: They have an escape velocity, but do they hover in space near the cathode?
: They are vigourously on the move all the time, coming and going, and the
: cloud density is slightly affected by the anode or screen voltages, and a lot
: affected by the
: g1 voltage; with a +ve going grid, electron stream away from the cathode, so
: the
: *net* emission increases, and it is a substantially linear phenomena.
:
:
:
: Reabsorbed? By what process?
: The heated cathode, boiling off electrons in a vacuum, is also reabsorbing
: them, you say?
:
: Nothing is "boiled off", but rather the electrons are given enough heat energy
: to
: spin off atoms, breaking the force which attracts them to the cathode.
: They slow down, and return, become re energised...
: Its like bees in a swarm, some are leaving and some come back, so a net
: effect is a cloud of bees on the move.
some snipped stuff

the boiling off of electrons, the earlier model i gave is quite accurate -
eg., electrons escaping the cathode actually have a cooling effect on
the cathode, just like in a boiling process.

in the absense of current, an equilibrium is reached, where the electrons
surrounding the cathode create a field opposing cathode electron escape.
as there is a spread of energy over cathode electrons, some will still escape,
others return from the space charge - analogous with a pressure vat with
boiling liquid.

Rudy

say, what about extending the idea of aligned G2 grid to minimize it's
current to the maximum: an isolated wire G2 grid, just for manipulation
of space charge ?
:
: Patrick Turner.
:
:
:
:


  #25   Report Post  
Posted to rec.audio.tubes
Chris Hornbeck
 
Posts: n/a
Default Grid #2 in ultralinear mode

On Sun, 22 Jan 2006 13:54:25 GMT, "Ian Iveson"
wrote:

The boiling analogy is poor because water doesn't fall back into the
kettle.


Not in open air, but an analogy that includes condensation
is actually pretty good near the cathode surface.

How fast do electrons pass through a vacuum?


Velocity is a function of voltage differences and
path length. In a 25 Kilovolt CRT electrons reach
velocities with relativistic mass gains of 5% or so.
Pretty fast.

Chris Hornbeck


  #26   Report Post  
Posted to rec.audio.tubes
Adam Stouffer
 
Posts: n/a
Default "mick" gets his BALLS cut OFF

Phil Allison wrote:


** PUBLIC WARNING TO MICK !!!


NEVER EVER POST on RAT again !

Or else I will ** FORCE ** you to **** off.


I *promise* that will be an extremely painful experience.

Autistic compewter ****s like you need to be publicly castrated.

With a blunt carving knife.

Much blood.

Much pain.


Sounds like terroristic threats to me.


Adam
  #27   Report Post  
Posted to rec.audio.tubes
Patrick Turner
 
Posts: n/a
Default Grid #2 in ultralinear mode



Ian Iveson wrote:

Patrick Turner wrote

Electrons swirl around the cathode in a cloud like a swarm of
bees,
and they are emitted and reabsorbed constantly by the cathode.


And positive electricity makes flowers grow...

OK, try reading the old books about thermionic emissions from
cathodes.


Why old? They didn't know so much then.


"Old books" means written before 1960, when ppl wrote the best
and most detailed books about tubes.
Then Soiled State started, and nobody bothered with tubes until they
became fashionable again,
and the latter day authors mainly set out to please audiophile DIY
types.

Most modern authors merely re-gurgitate what is in all the old books.



Anyway, it should be obvious that you could reply to *any* question
here with "read a book". Equally, you may assume that most ppl know
what a book is, and what it is for. Hence you may conclude that
"read a book" will but rarely be a useful answer, particularly if
you don't say which parts of which book.


I suggest you read some old books.

You seem to have more time to do so. I am very busy making amps
for ppl and repairing an re-wiring old amps.

Instead of telling me its wrong for me to tell you to read books,
try reading books, then tell me what you have learnt, and we will
comment
on whether you have understood what you have read.

Expose yourself to public scrutiny please.
Its your duty as a real intellectual to do so.





Anyway, if you recite books all day, you might expect ppl will have
less incentive to read them.

I was rather hoping a living nuclear scientist might respond...fresh
insight is not an impossibility.

What are you here for?


To tell ppl they should learn more.

Please learn more Ian.





...They have an escape velocity,...


No, that's just what they don't have. Otherwise they would escape
:-(


Electrons emitted from a cathode are so energized with heat that they
are able to escape the force holding them in their orbits arond atoms,
and they spin
off, and sometimes get swept right away from the cathode because an
anode attracts them enough.




I was told at school that electrons in metals tend to have affinity
to the bulk, rather than to particular atoms. Like a sea of
electrons in an ion matrix. The atomic forces don't take
much escaping...although I suppose you could have a notion of an
escape velocity for the atomic forces, and then another for the
electrostatic forces. They achieve the former but not the latter.

None of which takes into account the surface structure of the metal,
which is different from that within.


Plenty of info about all this is in the old books that were written.

No need to discuss your school experience unless what you learnt
conveys an accurate and real idea about the process of emission.

At the school i went to electron emission was not part of the syllabus.

Had it been included, and had i wanted to know more, I may have asked
the teacher
where I would find out more, and he would have told me to go to the
library.

People hardly read any more, except the paper maybe.
They expect enlightenment about anything and everything to be possible
from a 20 minute session with a screen and mouse.

But tube technology development mainly all happened from 1903 to 1963,
it virtually ended 30 years before the Internet, the Land Of Bull**** Or
Wisdom ( LOBOW )
became mainstream.

My granpa said,

"If yer don't look, yer won't know"

And he didn't tell me where to look.

Patrick Turner





cheers, Ian


  #28   Report Post  
Posted to rec.audio.tubes
Ian Iveson
 
Posts: n/a
Default Grid #2 in ultralinear mode

Chris Hornbeck wrote

The boiling analogy is poor because water doesn't fall back into
the
kettle.


Not in open air, but an analogy that includes condensation
is actually pretty good near the cathode surface.


Mmmm, hmm, hmph, er...maybe

But...but...the necessary pressure vessel fills up with steam...and
the valve doesn't fill up with electrons.

OK they have equilibrium in common. Wondering what else...change of
state? Latent heat?

How fast do electrons pass through a vacuum?


Velocity is a function of voltage differences and
path length. In a 25 Kilovolt CRT electrons reach
velocities with relativistic mass gains of 5% or so.
Pretty fast.


Right...dimly remember encountering the eV...odd how the electron
gets its very own unit of speed. I remember when I asked "What's
that in mph?", much handwaving would ensue. That must have been
because of the change in mass, possibly at a time when it was no
more than a dark suspicion that something was amiss. Presumably
that's there to stop the electron travelling faster than light?

If I ask how fast electrons travel in a vacuum between cathode and
25kV anode, and the answer is 25keV, I'm not sure I feel
enlightened. Now I know about relativistic mass I feel much better
:-) Does that translate to a proportion of the velocity of light?

But wait! What's that about path length? Don't remember that bit. I
was encouraged to believe that the velocity depended only on the
potential difference, regardless of distance. This must be the mass
change again...so do electrons arriving at the centre of a TV tube
have a different speed to those at the corner? Or different mass? Or
both, with the one compensating for the other I guess? Do they take
longer to get there?

Also I wonder now if I should be considering velocity rather than
speed. Do they all arrive with the same velocity normal to the
screen, and so at different speeds depending on angle?

One thing I have learned from all this is that electron flow is
coherent in a valve, otherwise the screen couldn't be hidden in the
shadow of the grid. I have tried looking this stuff up, but I find
either what I already think I know, or stuff that appears pretty
unknowable. Once I get to relativity I wonder if I might be better
of with the "swarm of bees" hypothesis.

I notice in passing that the CRT transposes the names of screen and
anode, compared to the tetrode:-)

Us materialists are an awkward lot. Or rather, since it is so long
since I met another, (perhaps I am the last?), me materialist is
awkward. Clearly a vacuum is totally out of bounds. So, thinking
again, I quite like this steam thing. The electrons are stayed by
pressure of ether, into which they evaporate when excited, to which
they lose energy, and from which they condense when calm. Cool.

Thanks Chris.

cheers, Ian




"in message ...
On Sun, 22 Jan 2006 13:54:25 GMT, "Ian Iveson"
wrote:

Chris Hornbeck



  #29   Report Post  
Posted to rec.audio.tubes
Corne Janssen
 
Posts: n/a
Default Grid #2 in ultralinear mode


"RdM" wrote in message
...
Patrick Turner wrote in
. au:

Electrons swirl around the cathode in a cloud like a swarm of bees,
and they are emitted and reabsorbed constantly by the cathode.


Reabsorbed? By what process?


The process is called recombination.
When an electron is made free, due to thermal energy in the cathode, it
leaves an atom with one electron less so it becomes positive. Atoms strive
for energy equilibrium so if it's missing an electron it wants to absorb an
electron. When an electron is absorbed by an atom it's recombined hence the
name recombination.

boiling off electrons in a vacuum, is also reabsorbing
them, you say? What percentage of emitted electrons would you say are
reabsorbed,


This depends on the electrical field the electrons experience near the
cathode.
When there's a high negative field all the electrons will stay near the
cathode (anode current is zero) and recombine.
At any given time there are atoms freeing electrons to the electron cloud,
electrons travelling in the electron cloud and electrons that are being
recombined from the electron cloud to atoms in the cathode.
When there's a small positive field some electrons travel from the electron
cloud to the anode so there's some anode current. When there's a very high
positive field all electrons from the electron cloud travel to the anode
this is called saturation.

Corne






  #30   Report Post  
Posted to rec.audio.tubes
Chris Hornbeck
 
Posts: n/a
Default Grid #2 in ultralinear mode

On Mon, 23 Jan 2006 12:30:50 GMT, "Ian Iveson"
wrote:

But...but...the necessary pressure vessel fills up with steam...and
the valve doesn't fill up with electrons.


I'm only suggesting that it's more useful to say that
it *does* fill up with electrons. Without an anode and
various grids and handwaving, we still have a decent
light bulb.


But wait! What's that about path length? Don't remember that bit. I
was encouraged to believe that the velocity depended only on the
potential difference, regardless of distance.


Voltage differential exerts a force; force exerts an
acceleration, rather than a velocity per se.


An interesting discussion; especially in light of our
(human) total ignorance of what an electron *is*.
We haven't the faintest clue.

FWIW, John Wheeler, while at UTexas, proposed that there
is only one electron, and that it bounces back and forth
between the beginning and the end of time. This explains why
all electrons look alike, and is completely consistent
with all modern physics. I love to refer to this for its
beauty and breadth.

Thanks, as always,

Chris Hornbeck


  #31   Report Post  
Posted to rec.audio.tubes
Ian Iveson
 
Posts: n/a
Default Grid #2 in ultralinear mode


flipper wrote

But...but...the necessary pressure vessel fills up with
steam...and
the valve doesn't fill up with electrons.

OK they have equilibrium in common. Wondering what else...change
of
state? Latent heat?


Any analogy can be drawn to absurdity by expanding it past the
limited
scope the analogy intended to address, which is what you're trying
to
do with that one.


No. If an analogy only has one thing in common, it is a worthless
analogy. Of course it would not be an analogy if it were the same in
all respects...I'm not asking for that. More than one thing would
do.

Right...dimly remember encountering the eV...odd how the electron
gets its very own unit of speed.


Not really, it's a matter of unit convenience but, at any rate
(pun),
eV isn't velocity, it's energy.


Yes of course, that's why it seemed odd to use it to answer a
question about speed. Ask a straight question about an electron, and
you will rarely get a straight answer.

I remember when I asked "What's
that in mph?", much handwaving would ensue. That must have been
because of the change in mass, possibly at a time when it was no
more than a dark suspicion that something was amiss. Presumably
that's there to stop the electron travelling faster than light?

If I ask how fast electrons travel in a vacuum between cathode and
25kV anode, and the answer is 25keV, I'm not sure I feel
enlightened. Now I know about relativistic mass I feel much better


eV is energy and (kinetic)E= .5mv^2


Speed or velocity? I raise the point because we normally think of
energy as scalar, and voltage as a vector.

Anyway, yes of course, that's why I said I felt much
better...because given the energy, I thought I also needed the mass
to work out the speed or velocity.

eV is convenient for particle physics because it is also the case,
for
this purpose at least, that E=MC^2. Energy has the units
(mass)(length)^2/(time)^2 so divide by C and you get momentum.
Divide
by C again and you get mass (old style "relativistic mass"). Lay
confusion stems from the particle physicists use of shorthand
'apparently' calling all three eV with the divide by C
'understood'
when speaking of momentum and by C^2 'understood' when speaking of
mass when, to be precise, it is eV/c and eV/c2 respectively.


Yes, this is the kind of loose usage I was complaining about. Also I
am pretty much bound to object to this changing mass lark but I'm
stuck for an alternative. Stalin killed our scientists before we
found out :-(

Path length is implicit in the eV term. Place a charge on two
plates
so you have a particular eV. Change the spacing and eV changes.


What? Run that past me again please, slowly.

"Potential difference" is the energy required to move a charged
particle from one potential point to the other against the applied
electric field. Let it 'free fall', I.E. be accelerated by that
field,
and it acquires the energy.


Right.

Which get's back to eV being energy.


What, again?

Also I wonder now if I should be considering velocity rather than
speed. Do they all arrive with the same velocity normal to the
screen, and so at different speeds depending on angle?

One thing I have learned from all this is that electron flow is
coherent in a valve, otherwise the screen couldn't be hidden in
the
shadow of the grid.


You don't need 'coherency' to cast a shadow and all one need do is
look at their own shadow on a sunny day to confirm it as sunlight
isn't even monochromatic, much less coherent.


You most certainly do need coherence to cast a shadow. Perhaps you
have a specialised meaning of coherence in mind...possibly germain
to this subject...like phase coherence, in which case I apologise
for misleading you.

I just mean coherent in the loose sense of an orderly structure. In
this case that paths are fairly parallel. Although sunlight is also
near-as-dammit parallel, it is not quite coherent in the sense I
mean. But close, which is why it casts sharp shadows.

The cathode of a typical valve is not even nearly a point source,
like the sun. And yet it casts shadows. That is all I meant by
coherent.

I have tried looking this stuff up, but I find
either what I already think I know, or stuff that appears pretty
unknowable. Once I get to relativity I wonder if I might be better
of with the "swarm of bees" hypothesis.
I notice in passing that the CRT transposes the names of screen
and
anode, compared to the tetrode:-)

Us materialists are an awkward lot. Or rather, since it is so long
since I met another, (perhaps I am the last?), me materialist is
awkward. Clearly a vacuum is totally out of bounds.


A vacuum is simply a place devoid of materialists

So, thinking
again, I quite like this steam thing. The electrons are stayed by
pressure of ether,


no


Oh, don't be such a spoilsport...why not?

into which they evaporate when excited,


They acquire enough energy to escape the electrical forces in the
atomic structure.


Atomic, molecular, electrostatic, whatever...it's metal. Easier to
say they acquire enough energy to part significantly from the bulk
material. I was trying to stick to the steam analogy anyway...you
are miles from there now.

to which
they lose energy,


Energy decay by quantum emission.


Oi! I'm a materialist. No quantum nonsense please. Of a photon? What
triggers that, or is it a statistical thing? This is nothing at all
like steam! Not keen on probabalistic explanations either. Are you
sure they simply don't achieve escape velocity?

and from which they condense when calm. Cool.


In the lower energy state they are recaptured by the positively
charged atomic structure returning everything to the neutral state
in
the absence of applied energy.


Again, I don't go along with your atomic structure thing. It's
metal...or is it something else? Do your electrons emit photons when
they are returned to their "atomic structure"? What frequency? IR?

Anyway, back to the original question, you say that the electron
loses its energy by quantum emission. That is an interesting
contribution, thanks. I will look it up with respect to the
structure of metals!

cheers, Ian



  #32   Report Post  
Posted to rec.audio.tubes
Ian Iveson
 
Posts: n/a
Default Grid #2 in ultralinear mode


"flipper" wrote in message
...
On Tue, 24 Jan 2006 01:56:54 GMT, "Ian Iveson"
wrote:


flipper wrote

But...but...the necessary pressure vessel fills up with
steam...and
the valve doesn't fill up with electrons.

OK they have equilibrium in common. Wondering what else...change
of
state? Latent heat?

Any analogy can be drawn to absurdity by expanding it past the
limited
scope the analogy intended to address, which is what you're
trying
to
do with that one.


No. If an analogy only has one thing in common, it is a worthless
analogy.


Nonsense. 'One thing' in common is perfectly fine if the purpose
is to
illuminate that 'one thing'.


Nope, since the "one thing" is already spoken for, there must be
other things to do the illumination. Another way of putting it, the
"one thing" analogy brings nothing new to the explanation, since
that one thing was there already. *All* the other things attached to
it become potentially misleading, extraneous baggage.

Of course it would not be an analogy if it were the same in
all respects...I'm not asking for that.


Near as I can tell you are. Regardless, you're trying to extend it
beyond it's applicability.


I am trying to find where its applicability lies, by process of
elimination.

More than one thing would
do.


What's relevant is whether the analogy works for the purpose it
was
made and, in that context, it does work fairly well.


As I agreed, once I re-jigged reality to fit :-)

Right...dimly remember encountering the eV...odd how the
electron
gets its very own unit of speed.

Not really, it's a matter of unit convenience but, at any rate
(pun),
eV isn't velocity, it's energy.


Yes of course, that's why it seemed odd to use it to answer a
question about speed. Ask a straight question about an electron,
and
you will rarely get a straight answer.


You misunderstood the 'answer' you were given. He said nothing
about
eV being 'speed' or 'velocity'. He said electrons accelerated
through
25KeV would reach relativistic speeds but that is no more a
statement
of eV *being* 'a speed' than someone pointing out you'll reach a
dangerous velocity falling off a cliff is some kind of claim that
the
earth's gravitational constant is 'a speed'.


No, not Chris, I meant what I learned at school. And what is still
in my school physics book. It involved an assumption about the mass
of the electron...I'm sure we all know the difference between speed
and energy.

Your analogy is false. The gravitational constant is not an
expression of energy. I can play that game too.

He merely said 25KeV is sufficient energy to do it.


Do what?

I remember when I asked "What's
that in mph?", much handwaving would ensue. That must have been
because of the change in mass, possibly at a time when it was no
more than a dark suspicion that something was amiss. Presumably
that's there to stop the electron travelling faster than light?

If I ask how fast electrons travel in a vacuum between cathode
and
25kV anode, and the answer is 25keV, I'm not sure I feel
enlightened. Now I know about relativistic mass I feel much
better

eV is energy and (kinetic)E= .5mv^2


Speed or velocity? I raise the point because we normally think of
energy as scalar, and voltage as a vector.

Anyway, yes of course, that's why I said I felt much
better...because given the energy, I thought I also needed the
mass
to work out the speed or velocity.


I have no idea what you're trying to complain about. Electron rest
mass is known, at least well enough.


You have lost the original sense. I notice you are not answering my
questions BTW. You are skipping them to niggle about your
misunderstanding of what I am saying. I was complaining about the
use of eV as an expression of speed. Mass is required for the
conversion. Problems ensue unless mass is a variable, however, for
otherwise, given a great enough potential, an electron would exceed
the speed of light.

Here, for me, the whole shebang becomes a matter of definition...of
compliance with other definitions...of coherence. Hope of absolute
truth is lost.

eV is convenient for particle physics because it is also the
case,
for
this purpose at least, that E=MC^2. Energy has the units
(mass)(length)^2/(time)^2 so divide by C and you get momentum.
Divide
by C again and you get mass (old style "relativistic mass"). Lay
confusion stems from the particle physicists use of shorthand
'apparently' calling all three eV with the divide by C
'understood'
when speaking of momentum and by C^2 'understood' when speaking
of
mass when, to be precise, it is eV/c and eV/c2 respectively.


Yes, this is the kind of loose usage I was complaining about.


Well, you can either complain or understand it and I suggest that
the
latter is more productive than expecting all the physicists in the
world to alter their common behavior.


Eh? But you were complaining about it...I just agreed with you.

I like to complain *and* understand. The two seem to go together
quite nicely.

As for all the physicists in the world, you are beginning to suffer
from Turneresque delusions. You haven't a clue what all physicists
think. Consensus, maybe...although a rather ragged one. Anyway,
physicists are paid to question theory, not agree with it.

Even if everything about the electron is agreed by "scientists" (why
assume I am not one, BTW, and what about philosophers...don't they
get a look in?), then surely you would wish that consensus remain
open to question? Isn't that what science is supposed to be?

Also I
am pretty much bound to object to this changing mass lark but I'm
stuck for an alternative.


You're not alone and modern physics doesn't use that 'analogy'
for
much, any more, other than for the purpose of an analogy, which is
why
I added the (old style "relativistic mass").


So what happens now, in your world, to stop an electron accelerated
through a large potential from exceeding the speed of light?

Stalin killed our scientists before we
found out :-(


Should have been plenty of time to grow new ones by now.


Where? Science requires community, not just to ensure sufficient
diversity for evolution, but also to protect itself from
persecution. Materialism is still a politically unacceptable
standpoint for the dominant ideology.

Path length is implicit in the eV term. Place a charge on two
plates
so you have a particular eV. Change the spacing and eV changes.


What? Run that past me again please, slowly.


Make a capacitor with movable plates. Charge it to a particular V.
Move the plates further apart or closer together. V changes.

Put wires on the plates and mound it in a cylinder suitable for
hand
holding and you have a condenser microphone.


Eh? You said eV changes...we were talking about the energy of
electrons passing from one electrode to the other, remember? Like in
a thermionic valve. You said path length is "implicit in the eV
term". Now you have dropped the "e". That's a bit naughty of you I
think.


"Potential difference" is the energy required to move a charged
particle from one potential point to the other against the
applied
electric field. Let it 'free fall', I.E. be accelerated by that
field,
and it acquires the energy.


Right.

Which get's back to eV being energy.


What, again?


eV is energy.


And again...


Also I wonder now if I should be considering velocity rather
than
speed. Do they all arrive with the same velocity normal to the
screen, and so at different speeds depending on angle?

One thing I have learned from all this is that electron flow is
coherent in a valve, otherwise the screen couldn't be hidden in
the
shadow of the grid.

You don't need 'coherency' to cast a shadow and all one need do
is
look at their own shadow on a sunny day to confirm it as
sunlight
isn't even monochromatic, much less coherent.


You most certainly do need coherence to cast a shadow. Perhaps you
have a specialised meaning of coherence in mind...possibly germain
to this subject...like phase coherence, in which case I apologise
for misleading you.


I see, well, when delving into the realm of particle physics,
especially in this day and age of lasers, people are likely to
think
you mean to include phase coherency when you toss out "coherent."

I just mean coherent in the loose sense of an orderly structure.
In
this case that paths are fairly parallel. Although sunlight is
also
near-as-dammit parallel, it is not quite coherent in the sense I
mean. But close, which is why it casts sharp shadows.


Ok. Now I know what you meant by it.

The cathode of a typical valve is not even nearly a point source,
like the sun. And yet it casts shadows. That is all I meant by
coherent.


It doesn't need to be a 'point source' because the plate isn't a
'point' either and the order comes from them being attracted to
the
plate.


Yes, but I think there is more to it than that.

They want to go ---- that away
---- that away
---- that away
---- that away


Er...:-) LOL !

Yes, that's probably what I mean :-) I guess the strongest field is
in the direction of the nearest point...although this is a rather
idealised view otherwise I suspect "hot spots" would be more common.
But there are magnetic effects too, and the tendency for electrons
to repel each other.

and if they make it past the grid they'll have a clear shot (give
or
take a few secondary forces) if the screen wires are in the same
'path' as the grid wires were.


Of course, as long as paths are parallel...or near enough for the
shadow to be significant. I think it's pretty blurred actually, for
all sorts of reasons, including diffraction.

I have tried looking this stuff up, but I find
either what I already think I know, or stuff that appears pretty
unknowable. Once I get to relativity I wonder if I might be
better
of with the "swarm of bees" hypothesis.
I notice in passing that the CRT transposes the names of screen
and
anode, compared to the tetrode:-)

Us materialists are an awkward lot. Or rather, since it is so
long
since I met another, (perhaps I am the last?), me materialist is
awkward. Clearly a vacuum is totally out of bounds.

A vacuum is simply a place devoid of materialists

So, thinking
again, I quite like this steam thing. The electrons are stayed
by
pressure of ether,

no


Oh, don't be such a spoilsport...why not?


Einstein


What...didn't he believe in the ether? When did he say that?

Do you believe in vacuums?

into which they evaporate when excited,

They acquire enough energy to escape the electrical forces in
the
atomic structure.


Atomic, molecular, electrostatic, whatever...it's metal. Easier to
say they acquire enough energy to part significantly from the bulk
material.


If you like "bulk material" better than "atomic structure," ok.

I was trying to stick to the steam analogy anyway...you
are miles from there now.


Not at all. When boiling water the water molecules acquire enough
energy to escape the attractive forces in the "bulk material." So
do
the electrons.


Oi! My "bulk material" isn't far from the analogy, I know that. Your
"atomic structure" was.

But, since electrons are negatively charged atomic particles of
extremely low mass, and water molecules are not, that's about as
far
as the analogy goes. But then, that is as far as it is intended to
go.


No, I like it now! The whole pressure vessel and all...fits with the
ether ;-)

to which
they lose energy,

Energy decay by quantum emission.


Oi! I'm a materialist. No quantum nonsense please. Of a photon?
What
triggers that, or is it a statistical thing?


What 'triggers' a hot body to radiate?

This is nothing at all
like steam!


Why do you have no trouble accepting that 'hot' steam will release
energy but do with electrons releasing energy?


Release of energy by hot steam is not a necessary part of the
analogy, if it is in an insulated pressure vessel. New steam stops
forming when a surface equilibrium is reached between pressure and
temperature. But you are taking the analogy too far now anyway.

The issue is whether they release energy, or merely convert it from
kinetic to potential, and back again.

Not keen on probabalistic explanations either.


Well, I think Schroedinger drew inappropriate conclusions from the
use
of statistics but that doesn't alter the mathematics of it.


It was Popper that ****ed me off.

Are you
sure they simply don't achieve escape velocity?


I don't know which 'they' you are talking about but if it used to
be
'locked' to --- here and is now flying off to -- there then
saying
'they' reached 'escape velocity' sounds like a reasonable analogy.


My little picture is just that the electrons are flung out at less
than the velocity required to escape the electrostatic field, so
they fall back. They don't need to loose energy to do that. OTOH, if
they must be ripped out of an atomic structure, I would expect they
would always achieve enough velocity to escape the field, and would
have to lose energy in order to return. But I'm still hung up on the
surface thing...surfaces are a world of their own. Perhaps the test
is whether they emit photons in the process. Does the ether glow?

Again, I don't go along with your atomic structure thing. It's
metal...or is it something else?


Surely you are aware that metals have an atomic structure. All
elements and compounds do.


Vaguely, ideally, sometimes, in varying degrees. Each metal can have
loads of different structures depending on impurities and formation.
The kind of metal you need to achieve decent perveance presumably
has very little structural energy invested in "outer" electron
positions. Not many materials make suitable cathodes, remember.
Cathode poisoning adds structure because compounds are formed, and
this stops emission.

Do your electrons emit photons when
they are returned to their "atomic structure"?


Close enough.


Eh?

What frequency? IR?


Depends on the energy state they were in and I'm not *that* good
enough a quantum mechanic to calculate it.


Well it doesn't appear to be visible light...and presumably it's not
dangerous.

Anyway, back to the original question, you say that the electron
loses its energy by quantum emission. That is an interesting
contribution, thanks. I will look it up with respect to the
structure of metals!


Doesn't matter if it's metal or anything else. An electron is an
electron and electrons acquire/release energy by
absorption/emission
of quantum packets or, the synonym, photons.


But of course it matters. Not every material makes a good cathode.
My question is whether a change of "energy state" is necessarily
involved, or whether it is just a change in kinetic energy.

They can also acquire kinetic energy, and lose it, without emitting
photons?

cheers, Ian


  #33   Report Post  
Posted to rec.audio.tubes
Ian Iveson
 
Posts: n/a
Default Grid #2 in ultralinear mode

Chris Hornbeck wrote

But...but...the necessary pressure vessel fills up with
steam...and
the valve doesn't fill up with electrons.


I'm only suggesting that it's more useful to say that
it *does* fill up with electrons. Without an anode and
various grids and handwaving, we still have a decent
light bulb.


As long as it's full of something :-)

But wait! What's that about path length? Don't remember that bit.
I
was encouraged to believe that the velocity depended only on the
potential difference, regardless of distance.


Voltage differential exerts a force; force exerts an
acceleration, rather than a velocity per se.


Yes, sure, but the force depends on voltage *and* distance, so
distance cancels. Kinetic energy is independent of distance, hence
the eV, which doesn't include distance. With constant mass, only the
voltage determines final velocity. Er...I've missed something
haven't I :-(

An interesting discussion; especially in light of our
(human) total ignorance of what an electron *is*.
We haven't the faintest clue.


The cave, etc. Faith, utility, blah-de-blah.

FWIW, John Wheeler, while at UTexas, proposed that there
is only one electron, and that it bounces back and forth
between the beginning and the end of time. This explains why
all electrons look alike, and is completely consistent
with all modern physics. I love to refer to this for its
beauty and breadth.


Wow, of course. Look what happened to me on the road to Damascus...

http://www.one-electron.com/

They must know...THE TRUTH!

cheers, Ian


  #34   Report Post  
Posted to rec.audio.tubes
Chris Hornbeck
 
Posts: n/a
Default Grid #2 in ultralinear mode

On Tue, 24 Jan 2006 12:46:24 GMT, "Ian Iveson"
wrote:

Voltage differential exerts a force; force exerts an
acceleration, rather than a velocity per se.


Yes, sure, but the force depends on voltage *and* distance, so
distance cancels. Kinetic energy is independent of distance, hence
the eV, which doesn't include distance. With constant mass, only the
voltage determines final velocity. Er...I've missed something
haven't I :-(


Not really, I'd guess. But electrostatic force is a square
function, and distance is linear, so they don't cancel,
in the usual sense of the word.

Wow, of course. Look what happened to me on the road to Damascus...

http://www.one-electron.com/


Not, *that's* funny.

Thanks, as always,

Chris Hornbeck
  #35   Report Post  
Posted to rec.audio.tubes
Ian Iveson
 
Posts: n/a
Default Grid #2 in ultralinear mode

I am saddened by your graceless and snide refusal to try and help me
with my circuit. Suddenly you are too much like hard work.

You are twisting, turning, talking rubbish and refusing to learn.

If you are so clever, how come you find yourself incapable of
circuit design, or even simulating valves? I can, you can't.

Since you say that you value consensus absolutely, and just in case
you may have misled the unwary, try typing "thermionic emission"
into google. Lest you find yourself incapable even of that, here are
a couple to start you off:

http://en.wikipedia.org/wiki/Thermionic_emission

http://www.john-a-harper.com/tubes201/

Note:

**"First of all, the energy of an electron corresponds directly to
its speed."

**Assuming constant mass, the final speed of an electron depends
only on the voltage through which it is accelerated, regardless of
distance.

**An electron thermionically emitted from a metal does not have to
escape atomic forces. It only has to escape the simple electrostatic
forces at the surface of the bulk material. Statistically, most
electrons from the cathode return by simple deceleration due to the
field. That is, they *fail* to achieve escape velocity. ***THERE IS
ONLY ONE ESCAPE*** It is not a two-stage leap. There is only one
escape velocity. They do not pass go, or collect 200UKP...no energy
state is changed. They do not emit photons as a result of a change
in energy level in the atom, because they are not in the atom in the
first place. Some electrons will always escape, and so the bottle is
always full of electrons. It is like steam emission in a perfectly
insulated pressure vessel, in this respect, but with no "latent
heat".

**Atoms are not simple and discrete as you imagine.

So now you try reading your precious consensus.

Worst of all, I have learned something and you haven't. Makes me
feel sick like parasite.

This is what I have learned. I was wrong to suggest that surface
structure necessarily weakens thermionic emission. I did however say
that I am hazy about surfaces. Less so now, thanks and no thanks.
Actually a layer of the right oxide can improve it greatly. This
currently appears to me a paradox, since I would expect electrons to
be more tightly bound in a compound. I wonder if the presence of the
oxide reduces the surface potential of electrons in the metal, or
whether the oxide itself is responsible for the emission. Now I will
go and find out.

Ian

"flipper" wrote in message
...
On Tue, 24 Jan 2006 11:59:40 GMT, "Ian Iveson"
wrote:


"flipper" wrote in message
. ..
On Tue, 24 Jan 2006 01:56:54 GMT, "Ian Iveson"
wrote:


flipper wrote

But...but...the necessary pressure vessel fills up with
steam...and
the valve doesn't fill up with electrons.

OK they have equilibrium in common. Wondering what
else...change
of
state? Latent heat?

Any analogy can be drawn to absurdity by expanding it past the
limited
scope the analogy intended to address, which is what you're
trying
to
do with that one.

No. If an analogy only has one thing in common, it is a
worthless
analogy.

Nonsense. 'One thing' in common is perfectly fine if the purpose
is to
illuminate that 'one thing'.


Nope, since the "one thing" is already spoken for, there must be
other things to do the illumination. Another way of putting it,
the
"one thing" analogy brings nothing new to the explanation, since
that one thing was there already.


A poor assumption as it took you a while to accept the 'one thing'
but
finally did, as you note below.


*All* the other things attached to
it become potentially misleading, extraneous baggage.


Which is the common fallacy of extending an analogy past it's
intended
applicability.

Of course it would not be an analogy if it were the same in
all respects...I'm not asking for that.

Near as I can tell you are. Regardless, you're trying to extend
it
beyond it's applicability.


I am trying to find where its applicability lies, by process of
elimination.


In the visualization of electrons 'boiling' off the cathode
because,
like the water molecules acquiring enough thermal energy to escape
what you prefer to call the "bulk material" the electrons also
acquire
enough energy from thermal excitation to escape the "bulk
material.".

More than one thing would
do.

What's relevant is whether the analogy works for the purpose it
was
made and, in that context, it does work fairly well.


As I agreed, once I re-jigged reality to fit :-)


Same reality. Acquired energy though thermal excitation.

Right...dimly remember encountering the eV...odd how the
electron
gets its very own unit of speed.

Not really, it's a matter of unit convenience but, at any rate
(pun),
eV isn't velocity, it's energy.

Yes of course, that's why it seemed odd to use it to answer a
question about speed. Ask a straight question about an electron,
and
you will rarely get a straight answer.

You misunderstood the 'answer' you were given. He said nothing
about
eV being 'speed' or 'velocity'. He said electrons accelerated
through
25KeV would reach relativistic speeds but that is no more a
statement
of eV *being* 'a speed' than someone pointing out you'll reach
a
dangerous velocity falling off a cliff is some kind of claim
that
the
earth's gravitational constant is 'a speed'.


No, not Chris, I meant what I learned at school. And what is still
in my school physics book. It involved an assumption about the
mass
of the electron...I'm sure we all know the difference between
speed
and energy.


Well, it was to his message that you pondered why the electron got
it's own units of 'speed' and his eV number was the only one
there, so
how is someone to know you mean something else?

Your analogy is false. The gravitational constant is not an
expression of energy. I can play that game too.


Ever hear of potential energy?

What you're demonstrating is the fallacy of simply looking for
something to argue about for the sake of arguing: the "I can play
that
game too" syndrome.

In the first place, the purpose of the analogy was to show
another,
but obvious, erroneous association and it would make no difference
to
the analogy whether it involved energy or anything else, just that
the
erroneous association was clear. However, as it turns out, the
'energy' aspect of it is quite analogous as well.

He merely said 25KeV is sufficient energy to do it.


Do what?


Don't you keep track of your own conversations?

He said "In a 25 Kilovolt CRT electrons reach
velocities with relativistic mass gains of 5% or so.
Pretty fast."


I remember when I asked "What's
that in mph?", much handwaving would ensue. That must have
been
because of the change in mass, possibly at a time when it was
no
more than a dark suspicion that something was amiss.
Presumably
that's there to stop the electron travelling faster than
light?

If I ask how fast electrons travel in a vacuum between cathode
and
25kV anode, and the answer is 25keV, I'm not sure I feel
enlightened. Now I know about relativistic mass I feel much
better

eV is energy and (kinetic)E= .5mv^2

Speed or velocity? I raise the point because we normally think
of
energy as scalar, and voltage as a vector.

Anyway, yes of course, that's why I said I felt much
better...because given the energy, I thought I also needed the
mass
to work out the speed or velocity.

I have no idea what you're trying to complain about. Electron
rest
mass is known, at least well enough.


You have lost the original sense. I notice you are not answering
my
questions BTW. You are skipping them to niggle about your
misunderstanding of what I am saying.


then be clear.

I was complaining about the
use of eV as an expression of speed.


eV is not used as an expression of 'speed'. It's energy.

Mass is required for the
conversion.


And the electrons rest mass is known.

Problems ensue unless mass is a variable,


Actually, the 'problems' ensue when mass *is* a variable.

however, for
otherwise, given a great enough potential, an electron would
exceed
the speed of light.


That is the 'Newtonian physics' style thinking which leads to the
flawed 'relativistic mass' analogy.

E=mc^2 is the rest energy due to rest mass m. For relativistic
speeds
the equation is E^2 = m^2c^4 + p^2c^2, where p is the momentum.
Newtonian momentum is p = mv. Relativistic momentum is p = gamma
(mv)

"Mass" does not change, the momentum does.

Here, for me, the whole shebang becomes a matter of
definition...of
compliance with other definitions...of coherence. Hope of absolute
truth is lost.


In quantum mechanics the search for 'absolute truth' runs into the
Heisenberg uncertainty principle.

eV is convenient for particle physics because it is also the
case,
for
this purpose at least, that E=MC^2. Energy has the units
(mass)(length)^2/(time)^2 so divide by C and you get momentum.
Divide
by C again and you get mass (old style "relativistic mass").
Lay
confusion stems from the particle physicists use of shorthand
'apparently' calling all three eV with the divide by C
'understood'
when speaking of momentum and by C^2 'understood' when
speaking
of
mass when, to be precise, it is eV/c and eV/c2 respectively.

Yes, this is the kind of loose usage I was complaining about.

Well, you can either complain or understand it and I suggest
that
the
latter is more productive than expecting all the physicists in
the
world to alter their common behavior.


Eh? But you were complaining about it...I just agreed with you.

I like to complain *and* understand. The two seem to go together
quite nicely.

As for all the physicists in the world, you are beginning to
suffer
from Turneresque delusions. You haven't a clue what all physicists
think. Consensus, maybe...


You're playing with semantics to no purpose. Of course I mean the
current 'consensus' and it isn't that hard to find. And whether
it's
'all" or 'all minus a few' doesn't alter the meaning of my
statement.
It's still more productive to understand what the 'consensus' is
than
expect 'all minus a few' to change their behavior.

although a rather ragged one.


It isn't 'ragged' on the elementals we're talking about here.

Anyway,
physicists are paid to question theory, not agree with it.


The discussion in this section isn't about 'theory', it's about
the eV
nomenclature.

Even if everything about the electron is agreed by "scientists"
(why
assume I am not one, BTW, and what about philosophers...don't they
get a look in?), then surely you would wish that consensus remain
open to question? Isn't that what science is supposed to be?


Science, at it's root, is experimentation and observation.
'Philosophy' ain't in there because it proposes no observable
experiment, or else it would be a theory and not 'philosophy'.

Schroedinger wandering off into the philosophical implications of
his
statistical quantum equations notwithstanding.

Also I
am pretty much bound to object to this changing mass lark but
I'm
stuck for an alternative.

You're not alone and modern physics doesn't use that 'analogy'
for
much, any more, other than for the purpose of an analogy, which
is
why
I added the (old style "relativistic mass").


So what happens now, in your world, to stop an electron
accelerated
through a large potential from exceeding the speed of light?


It ain't 'my world'. It's Einstein's actual equation.

See above.

Stalin killed our scientists before we
found out :-(

Should have been plenty of time to grow new ones by now.


Where? Science requires community, not just to ensure sufficient
diversity for evolution, but also to protect itself from
persecution. Materialism is still a politically unacceptable
standpoint for the dominant ideology.


If you think they got no physicists then so be it but I don't feel
like trying to unravel whatever that was.

Path length is implicit in the eV term. Place a charge on two
plates
so you have a particular eV. Change the spacing and eV
changes.

What? Run that past me again please, slowly.

Make a capacitor with movable plates. Charge it to a particular
V.
Move the plates further apart or closer together. V changes.

Put wires on the plates and mound it in a cylinder suitable for
hand
holding and you have a condenser microphone.


Eh? You said eV changes...


Right.

we were talking about the energy of
electrons passing from one electrode to the other, remember?


Most definitely.

Like in
a thermionic valve. You said path length is "implicit in the eV
term". Now you have dropped the "e". That's a bit naughty of you I
think.


1 eV is the energy given to an electron by accelerating it through
1
volt of electric potential difference. Charge two plates to V
potential difference, let the electron be accelerated from one to
the
other and it acquires eV energy.

Move the charged plates further apart or closer together and the V
changes, and so does the resultant eV of an electron accelerated
between them because the V is different.

The length becomes moot by specifying the plates are at a specific
'X'kV because you have placed on the plates whatever charge is
necessary to achieve the 'X'kV for the particular spacing. The
required charge (to created the energy field between the plates)
will
be more or less if the plate spacing is more or less so the
'length'
is implicit in the potential difference V you define as being on
the
plates and, hence, the resultant eV, which will be the same number
as
the V potential difference because it's a 1 to 1 calculation when
accelerating electrons since it's defined as the energy an
electron
acquires per V of potential difference. I.E. for an electron the
acquired eV is numerically equal to the potential difference V
it's
accelerated through, by definition.

e.g. Make 25kV plates and the electrons acquire 25keV, because
they're
electrons and that's how it's defined.

So there's nothing strange or 'naughty' about me using V when
speaking
of the V on the plates that cause the electrons to acquire eV.

"Potential difference" is the energy required to move a
charged
particle from one potential point to the other against the
applied
electric field. Let it 'free fall', I.E. be accelerated by
that
field,
and it acquires the energy.

Right.

Which get's back to eV being energy.

What, again?

eV is energy.


And again...


I don't know how to say it any simpler so if you're going to keep
repeating the question just reread what's there.

Also I wonder now if I should be considering velocity rather
than
speed. Do they all arrive with the same velocity normal to the
screen, and so at different speeds depending on angle?

One thing I have learned from all this is that electron flow
is
coherent in a valve, otherwise the screen couldn't be hidden
in
the
shadow of the grid.

You don't need 'coherency' to cast a shadow and all one need
do
is
look at their own shadow on a sunny day to confirm it as
sunlight
isn't even monochromatic, much less coherent.

You most certainly do need coherence to cast a shadow. Perhaps
you
have a specialised meaning of coherence in mind...possibly
germain
to this subject...like phase coherence, in which case I
apologise
for misleading you.

I see, well, when delving into the realm of particle physics,
especially in this day and age of lasers, people are likely to
think
you mean to include phase coherency when you toss out
"coherent."

I just mean coherent in the loose sense of an orderly structure.
In
this case that paths are fairly parallel. Although sunlight is
also
near-as-dammit parallel, it is not quite coherent in the sense I
mean. But close, which is why it casts sharp shadows.

Ok. Now I know what you meant by it.

The cathode of a typical valve is not even nearly a point
source,
like the sun. And yet it casts shadows. That is all I meant by
coherent.

It doesn't need to be a 'point source' because the plate isn't a
'point' either and the order comes from them being attracted to
the
plate.


Yes, but I think there is more to it than that.

They want to go ---- that away
---- that away
---- that away
---- that away


Er...:-) LOL !

Yes, that's probably what I mean :-) I guess the strongest field
is
in the direction of the nearest point...although this is a rather
idealised view otherwise I suspect "hot spots" would be more
common.
But there are magnetic effects too, and the tendency for electrons
to repel each other.


That's why I mentioned "secondary forces" below. But your sunlight
shadow comes out pretty good regardless of atmospheric diffusion,
and
'secondary forces', too.

and if they make it past the grid they'll have a clear shot
(give
or
take a few secondary forces) if the screen wires are in the same
'path' as the grid wires were.


Of course, as long as paths are parallel...or near enough for the
shadow to be significant. I think it's pretty blurred actually,
for
all sorts of reasons, including diffraction.


When did how 'wonderful' a shadow it is become the topic? and how
did
you arrive at the belief it's "pretty blurry?"

Never mind. It doesn't matter. I'm satisfied with their
"scientific
observation" (since that was a topic up there) that "hey, it
works."


I have tried looking this stuff up, but I find
either what I already think I know, or stuff that appears
pretty
unknowable. Once I get to relativity I wonder if I might be
better
of with the "swarm of bees" hypothesis.
I notice in passing that the CRT transposes the names of
screen
and
anode, compared to the tetrode:-)

Us materialists are an awkward lot. Or rather, since it is so
long
since I met another, (perhaps I am the last?), me materialist
is
awkward. Clearly a vacuum is totally out of bounds.

A vacuum is simply a place devoid of materialists

So, thinking
again, I quite like this steam thing. The electrons are stayed
by
pressure of ether,

no

Oh, don't be such a spoilsport...why not?

Einstein


What...didn't he believe in the ether? When did he say that?


I'll leave reading about Einstein to your research.

Do you believe in vacuums?


Define vacuum.

into which they evaporate when excited,

They acquire enough energy to escape the electrical forces in
the
atomic structure.

Atomic, molecular, electrostatic, whatever...it's metal. Easier
to
say they acquire enough energy to part significantly from the
bulk
material.

If you like "bulk material" better than "atomic structure," ok.

I was trying to stick to the steam analogy anyway...you
are miles from there now.

Not at all. When boiling water the water molecules acquire
enough
energy to escape the attractive forces in the "bulk material."
So
do
the electrons.


Oi! My "bulk material" isn't far from the analogy, I know that.
Your
"atomic structure" was.


All elements and compounds are an 'atomic structure' and, no, my
use
of it doesn't increase the analogy's 'distance'.


But, since electrons are negatively charged atomic particles of
extremely low mass, and water molecules are not, that's about as
far
as the analogy goes. But then, that is as far as it is intended
to
go.


No, I like it now! The whole pressure vessel and all...fits with
the
ether ;-)


no


to which
they lose energy,

Energy decay by quantum emission.

Oi! I'm a materialist. No quantum nonsense please. Of a photon?
What
triggers that, or is it a statistical thing?

What 'triggers' a hot body to radiate?

This is nothing at all
like steam!

Why do you have no trouble accepting that 'hot' steam will
release
energy but do with electrons releasing energy?


Release of energy by hot steam is not a necessary part of the
analogy,


A moot comment as you're trying to create a false analogy anyway.

if it is in an insulated pressure vessel.


It still looses energy, only the time constant is altered.

New steam stops
forming when a surface equilibrium is reached between pressure and
temperature. But you are taking the analogy too far now anyway.


No, you are as I specifically said this nonsense was not part of
the
original analogy.

The issue is whether they release energy, or merely convert it
from
kinetic to potential, and back again.


The electron's charge does not change, or else it isn't an
electron.

Not keen on probabalistic explanations either.

Well, I think Schroedinger drew inappropriate conclusions from
the
use
of statistics but that doesn't alter the mathematics of it.


It was Popper that ****ed me off.

Are you
sure they simply don't achieve escape velocity?

I don't know which 'they' you are talking about but if it used
to
be
'locked' to --- here and is now flying off to -- there then
saying
'they' reached 'escape velocity' sounds like a reasonable
analogy.


My little picture is just that the electrons are flung out


Because they have acquired energy from the thermal excitation.

at less
than the velocity required to escape the electrostatic field,


Which electrostatic field? The original low energy state field
when
they were bound in the material's atomic structure or the one
created
by them now removed from it thereby creating a positive potential
on
the material they escaped from?

so
they fall back. They don't need to loose energy to do that.


They most certainly do.

In your 'picture, how do you resolve conservation of energy with
energy needed to get them wandering around in the first place but
they'll 'go back' with it still there? Or, put another way, your
'picture' has the electron acquiring energy and saying "I'm
released!,
no I'm captured, no I'm released!, no I'm captured" all at the
same
time. Makes no sense.

If an electron is released by the acquisition of energy, which it
is,
then that energy has got to go away for it to be recaptured, or
else
it wasn't released, or else it isn't recaptured. But it can't be
both
at the same energy state.


OTOH, if
they must be ripped out of an atomic structure,


There is no "OTOH." All materials have an atomic structure.

I would expect they
would always achieve enough velocity to escape the field, and
would
have to lose energy in order to return. But I'm still hung up on
the
surface thing...surfaces are a world of their own.


What you're 'hung up' on is in attempting to deny that matter is
made
up of atoms and particles.

Particle physics doesn't 'change', nor 'go away', based on the
'material' involved or that it's on a 'surface'.

Perhaps the test
is whether they emit photons in the process.


They do.

Does the ether glow?


Define glow.

Again, I don't go along with your atomic structure thing. It's
metal...or is it something else?


Surely you are aware that metals have an atomic structure. All
elements and compounds do.


Vaguely, ideally, sometimes, in varying degrees. Each metal can
have
loads of different structures depending on impurities and
formation.
The kind of metal you need to achieve decent perveance presumably
has very little structural energy invested in "outer" electron
positions. Not many materials make suitable cathodes, remember.
Cathode poisoning adds structure because compounds are formed, and
this stops emission.


None of which has anything to do with the simple, basic, fact that
all
elements and compounds have an atomic structure.

Do your electrons emit photons when
they are returned to their "atomic structure"?

Close enough.


Eh?


Ew, e, ew ah ah. Bing bang walla walla bing bang.

Sorry, but that's all I know about "eh."

What frequency? IR?

Depends on the energy state they were in and I'm not *that* good
enough a quantum mechanic to calculate it.


Well it doesn't appear to be visible light...and presumably it's
not
dangerous.


Visible light is but a miniscule part of the EM spectrum and the
only
significance to it is our eyeballs like it. But electrons couldn't
care less.

Anyway, back to the original question, you say that the electron
loses its energy by quantum emission. That is an interesting
contribution, thanks. I will look it up with respect to the
structure of metals!

Doesn't matter if it's metal or anything else. An electron is an
electron and electrons acquire/release energy by
absorption/emission
of quantum packets or, the synonym, photons.


But of course it matters.


No, it doesn't.

Not every material makes a good cathode.


"What makes a good cathode" isn't the topic.

My question is whether a change of "energy state" is necessarily
involved, or whether it is just a change in kinetic energy.


How can you even say that? Kinetic ENERGY is ENERGY and a change
in
ENERGY is a change in the ENERGY state.

They can also acquire kinetic energy, and lose it, without
emitting
photons?


Not in this context (meaning, for one, we're not talking about a
particle smasher or matter-antimatter annihilation)

cheers, Ian




Reply
Thread Tools
Display Modes

Posting Rules

Smilies are On
[IMG] code is On
HTML code is Off


Similar Threads
Thread Thread Starter Forum Replies Last Post
VFD as an audio/RF amplifier? N9WOS Vacuum Tubes 5 August 2nd 05 10:48 AM
Grid resistance for SET 2A3 Gilbert Bates Vacuum Tubes 7 June 22nd 05 11:51 AM
KISS 121 by Andre Jute [email protected] Vacuum Tubes 0 April 22nd 05 09:30 PM
Some tube history about 6L6. Patrick Turner Vacuum Tubes 10 August 28th 04 06:24 PM
Grid current in 6C33C John Harper Vacuum Tubes 4 August 22nd 03 12:15 PM


All times are GMT +1. The time now is 07:11 PM.

Powered by: vBulletin
Copyright ©2000 - 2024, Jelsoft Enterprises Ltd.
Copyright ©2004-2024 AudioBanter.com.
The comments are property of their posters.
 

About Us

"It's about Audio and hi-fi"